ImageVerifierCode 换一换
格式:DOCX , 页数:56 ,大小:326.85KB ,
资源ID:3527458      下载积分:3 金币
快捷下载
登录下载
邮箱/手机:
温馨提示:
快捷下载时,用户名和密码都是您填写的邮箱或者手机号,方便查询和重复下载(系统自动生成)。 如填写123,账号就是123,密码也是123。
特别说明:
请自助下载,系统不会自动发送文件的哦; 如果您已付费,想二次下载,请登录后访问:我的下载记录
支付方式: 支付宝    微信支付   
验证码:   换一换

加入VIP,免费下载
 

温馨提示:由于个人手机设置不同,如果发现不能下载,请复制以下地址【https://www.bdocx.com/down/3527458.html】到电脑端继续下载(重复下载不扣费)。

已注册用户请登录:
账号:
密码:
验证码:   换一换
  忘记密码?
三方登录: 微信登录   QQ登录  

下载须知

1: 本站所有资源如无特殊说明,都需要本地电脑安装OFFICE2007和PDF阅读器。
2: 试题试卷类文档,如果标题没有明确说明有答案则都视为没有答案,请知晓。
3: 文件的所有权益归上传用户所有。
4. 未经权益所有人同意不得将文件中的内容挪作商业或盈利用途。
5. 本站仅提供交流平台,并不能对任何下载内容负责。
6. 下载文件中如有侵权或不适当内容,请与我们联系,我们立即纠正。
7. 本站不保证下载资源的准确性、安全性和完整性, 同时也不承担用户因使用这些下载资源对自己和他人造成任何形式的伤害或损失。

版权提示 | 免责声明

本文(高级宏观经济学第四版中文罗默课后题答案.docx)为本站会员(b****5)主动上传,冰豆网仅提供信息存储空间,仅对用户上传内容的表现方式做保护处理,对上载内容本身不做任何修改或编辑。 若此文所含内容侵犯了您的版权或隐私,请立即通知冰豆网(发送邮件至service@bdocx.com或直接QQ联系客服),我们立即给予删除!

高级宏观经济学第四版中文罗默课后题答案.docx

1、高级宏观经济学第四版中文罗默课后题答案高级宏观经济学第四版中文罗默课后题答案第2章无限期模型与世代交叠模型2.1 考虑N个厂商,每个厂商均有规模报酬不变的生产函数 心F(K/厶),丫“(K,乩),或者采用紧凑形式心仙W。假设 /( *) ,厂() 0,则积分项收敛,为l/(P -n),则:(7)将方程(7)代入(4):C(t) = pt為(P n)(8)因此,初始消费为:(9)个人的初始财富为厶(。)加,方程(9)说明消费是初始财富的一个不变的比例。(P-n)为个人的财富边际消费倾向。可以看出,这个财富边际消费倾向在平衡增长路径上是独立于利率的。对于折现率P而言,P越大,家庭越厌恶风险,越会选

2、择多消费。2.5设想某家庭的效用函数由(2.1) (2.2)式给定。假设实际利率不 变,令伊表示家庭的初始财富加上终生劳动收入的现值(2. 6)的右端。已知二 罗和效用函数中的各参数,求C的效用最大化路径。 =J 二 盯叫(C()罟dt 2.1u(C(Q)=需 2. 2答:本题目是在家庭的预算约束下最大化一生的效用,即:max =二0广S(C(t)營r (1)(2)伊代表家庭的初始财富加上家庭一生劳动收入的现值,利率/是常数。建立拉格朗日方程如下:求一阶条件,可得:0LdC(i)o(0=0eptC(t) = Ae-rt (3)两边对时间t求导,可得:ept- o C(t)- pe-ptC(t)

3、_ 0 + r Xe_rt= 0得到下面的方程:-ptC(ty 6 - peptc 0 +rAert=O (4)将方程(3)代入(4),可得:-9 - Xert p Xert + r Xe_rt = 0C(t)rr (0抵消入 然后求消费的增长率丽,可得:(5)C(巧 _ r- 由于利率/是常数,所以消费的增长率为常数。如果r P,则市场利率超过贴现率,则消费会增加;反之,如果厂VP,则市场利率小于贴现率,则消费会减少。如果rP,则决定了消费增长的幅度。值越低,也就是替代弹性越高,越髙,即消费增长的越快。重写方程(5),得:(6)dlnC(t) r - pdt= 8对方程(6)积分,积分区间是

4、从时间了二0到时间r=t,可得:加C(%C(0)=宁 r| 上式可以简化为:/nC(t)/C(O) = (r- p)/ Ot(7)对方程(7)两边取指数,可得:C(t)/C(0) = (rP)/K,整理得:C(t) = C(0)e(r- P)/(8)下面求解初始消费,将方程(8)代入(2),可得:将L(t) = emL(O)代入上式,可得:“0廿(0)二詳-Q7+ 必=(9)只要P_厂+ O(r-n)/O 0,从而保证积分收敛,则求解方程(9)可得:f8 -p -r+ 心 7加 =J C = 0 p -r+ W-n)(10)将方程(10)代入(9)中,求解0):W、 ” W) c()将方程(i

5、d代入(8),求解ca):C(f) 爲字+() (12)上式便是C的效用最大化路径。2.6生产力增长减速与储蓄。设想一个正处于平衡增长路径上的拉姆塞一 卡斯一库普曼期模型,假设g永久性下降。(a) = 曲线会如何变化(如果有形响)?(b)c = 曲线会如何变化(如果有影响)?(c)当g下降时,c如何变化?(d)用一个式子表示g的边际变化对平衡增长路径上储蓄率的影响。能否 判断此表达式的正负?(e)设生产函数長柯布一道格拉斯函数= 请用P、从&、0和a 重新表示(d)中的结果。(提示:利用等式fd= p + 9。)答:(a)关于资本的欧拉方程为:k(t) = /c(t) -c(t) -(n +

6、( 1)该方程描述了资本的动态方程,在拉姆塞模型中,该方程描述了技术特征, 是该模型的核心,它与消费的动态方程一起构成了该模型的欧拉方程组,从而决 定了该模型的最终解。在平衡增长路径上,“二0,由此可以推出:c(k)-(n + g)k。在该方程 中,当g永久性地下降时,会导致消费c上升以保持方程的均衡。因而在图形上 k = 0曲线向上移动。同时,保持斤不变,g永久性地下降会导致持平投资下降, 这样就会有更多的资源用于消费。由于持平投资+ 0)下降的幅度更大,因而 在更高的斤水平上,二向上移动得更大。图2-1是该模型的图示。(b)每单位有效劳动消费的欧拉方程为:- p - 9 该方程描述了消费的

7、动态方程,在拉姆塞模型中,该方程描述了偏好特征, 是该模型的核心,它与资本的动态方程一起构成了该模型的欧拉方程组,从而决 定了该模型的最终解。在平衡增长路径上,要求c二0,即fW = P + 00,在g永久性地下降时, 为保持E二0,才住)必须下降。由于/(/) 0,因而下降必然导致R上升。 因此,二必须上升,在图形上表现为c二向右移动,如图2-1所示。(O在g永久性地下降时,由于每单位有效劳动的资本是由历史上的投资 决定的,因而不会发生不连续的变化。它仍然保持在平衡增长路径以处。与此相反,每单位有效劳动的消费则会随着g永久性地下降而迅速变化。为 使经济从旧的平衡增长路径达到新的平衡增长路径,

8、每单位有效劳动的消费C 必将发生变化。不过,此处无法确定新的平衡增长路径处于旧的均衡点的上边还是下边,因 而无法确定每单位有效劳动的消费C是上升还是下降。存在一种特殊情况,即如 果新的平衡增长路径恰好位于旧的均衡点的右上方,则每单位有效劳动的消费C 甚至可能保持不变。因此,c和&逐步移动到新的平衡增长路径,此时的值高于 原先的平衡增长路径值。(C1)在平衡增长路径上,产出中被储蓄的部分为:fw c * 伙因为斤保持不变,即二,位于一条均衡的增长路径上,则由方程(1)可 知:fg -c* = O + 9)/r由上面两个式子可以推出在平衡增长路径上,产出中被储蓄的份额为:s = (n + g)k

9、/f(k *)对方程(3)两边关于g求导数,可得:ds _ f(k *)(n + g)dk 伽 + L - (n + g)k,f(k *)d/c * /dg乔 砒)2可以再简化为:ds _ 5 + 丁0“)(就/0g) + fX)T爾_ imp由于疋由f (k) = p +切决定,对该式两边关于g求导数, f(k)(b/g)= 0,从而求出上*/g为:/厂理)0将方程(5)代入(4)中,可得:ds _ (n + g)fX)-kf) 0 +f理片广X)(3)(4)可得:(5)(6)在方程(6)中,分母r(4)2/(3为负,分子中第一项为正,而第二项 为负,因而无法确定正与负。因此,无法判断在平衡

10、增长路径上g永久性地下降 会使S上升还是下降。(e )将柯布一道格拉斯生产函数f(k) = k“ , f(k) = ak 7和 f(k)= a (a -l)ka2代入方程(6)中,可得:ds (n + )/c* Q -/c+ a k* Q _1 0 + / * Q /c+ a ( a -l)/c* a _2乔 k*ak*ua(a -l)/c*u2简化为:_ (n + )/c4 Q (1 - a ) e -(1- a)/c* a a/c*G _1矿 -(1- u)/0的情况时,消费和资本的欧拉方程变为:c(t) - i- p- OQ(1)kt) = /(/c(t) - c(t) - (n + “

11、 + (2)对方程(1)和(2)分别在c = 和上二L处进行一阶泰勒展开,可得:(4)定c = c-c4和k = ,因为C*和L为常数,所以c = c且 将(3)和(4)重写为:对方程(1)和(2)计算偏导数:dedk bgp = Tbgp6_ p_ ffg(5)(6)(7)(8)專仏= f(b)- + 9+ 6 (9)bgp = 1 (10)将方程(7)和(8)代入(5),将方程(9)和(10)代入(6),可得:C = -k (11)k = f k ) _ + g + 一 C=( + Q + Og) 一( + 9 + S)k - c=k-c (12)方程(12)的第二步用到了/(fet) =

12、(6 + P + 9),第三步用到了定义0 = P 0)9。对方程(11)除以2以求$的增长率,对方程(12)除以&以求&的增长率:(13)(14)可以发现该结果与教材中不存在折旧率的增长率一样,也就是说折旧率的存在对增长率没有影响。因此,经济在向平衡增长路径移动时的和*的不变增长率与教材中的结果应该一致。C(15)_ dr=,求解可令卩=;方程(13)可以推出:由方程(15),令(13)和(14)相等,可得: 得:B 32-4(/c*)c701/2M 二 2 如果卩为正,则经济会偏离稳定点,所以必为负:P -p2-4/(k*)c7e1/2现在考虑柯布一道格拉斯生产函数代k) = k“,分别求

13、其一阶导和二阶导:f (/J = al 71 =r* + 8(16)(17)/ (Ar*)= a/T 一1= a(a _)L 一2/ 、2 21 2 2将方程(16)两边同时平方:(r + 5 ) = a /c ,将其代入(17)式:茁)少 + 从)(-1)(r,+8)2 (1 a kG 心)定义平衡增长路径上的储蓄率为,则平衡增长路径上的消费为:(18)将方程(17)和(18)代入(15):9 ( a - 1)(厂* + 5)2P 2 4 (1 一 s (/T )化简为:护+岁磐(厂+ )2(17)= 2 (19)在平衡增长路径上, = 意味着厂=P + 即:厂 + / = q + 8 g

14、+ (5(20)可以推出:(21)(22)另外,实际投资等于持平投资:sJ(L) = (n + g+4 (n + g + ff)k * a (n + g +_ f(/r) _ k上步用到了八+ 5 = k4a由(21)可以推岀:r* + 6 a(n + g + /) _S = r- +将方程(20)和(22)代入到(19)中,可得:fi 一+ + 0+ “(n + 0+ 6)U = 2 上式与教材中的(2.39)极其相似,它表明了消费与资本的调整速度(将a = 1/3 , p = 4% n = 2% = 1% , 0=1, = 3% 代入上式,得到1= 8-8%)要快于不存在折旧时的调整速度。

15、2.9拉姆塞模型的解析解来自于史密斯(Smith,2006)。考虑生产函数柯 布-道格拉斯函数的拉姆塞模型,y(o = 1的情形,假设相对风险规避系数 与资本份额a相等。(a)平衡增长路径上的值(即住* )为多少?(b)平衡增长路径上的c值(即& * )为多少?(c)令呗)表示资本产出比班)表示消费资本比请用 z、X和模型参数表示z(t)和兀()/兀0)。(d)暂且猜測才在鞍点路径上是常数,根据这一猜想:(i)给定初始值。),求Z的路径。(ii)给定初始值做。),求y的路径。经济沿鞍点路径向平衡增长路径收 敛的速度是否是常数?(e)上述猜测的解是否满足c与k的运动方程(2. 24)与(2. 2

16、5) ?答:(a)已知从正文可知,在c = 0时,存在f(k)= P + 00。利用方程(1)计算得到(b )与题类似,根据正文可知,在k = 0时,存在 c* + 利用方程(1)计算得到:(C)设Z(t) = 和XO = c(t)/fc(Oo将方程(!)代入z(t)的定义得将方程(4)代入只)的定义,得到:(5)ck- a =xz(7)(8)(9)(10)(11)(12)使用方程(4),考虑z(t) = 的时间导数,得到:z= (1- a* ak (6)从正文.的方程(2. 25)知道,k =ka-c-(n- 方程(6)可表示成:z = (1 - ka - c - (n + g)k为简化上式,将方程(4)和方程(5)代入上式,得到:Z = (1 - ) 1 - XZ -(71 + 9)z现在,对数化XO = c(tWO,考虑其时间导数,得到: x c k=x c k根据正文的方程(2.24)和方程(2.25),上式可表示成:x Qk P _ Og - k+ c + (?i + g)kX = L + k 将方程(4)和方程(5)代入上式,再利用a = 得到:一=兀 + 72 pj aX(d)(i)根据为常量的假设,方程(8)可表示成 z = (1 - a)l- (n + g + x*)z为确定z的变化路径,考虑方程(12)方程(12)为线性非齐次常微分方程。 该方程的

copyright@ 2008-2022 冰豆网网站版权所有

经营许可证编号:鄂ICP备2022015515号-1